Last visit was: 11 Jun 2024, 12:02 It is currently 11 Jun 2024, 12:02
Close
GMAT Club Daily Prep
Thank you for using the timer - this advanced tool can estimate your performance and suggest more practice questions. We have subscribed you to Daily Prep Questions via email.

Customized
for You

we will pick new questions that match your level based on your Timer History

Track
Your Progress

every week, we’ll send you an estimated GMAT score based on your performance

Practice
Pays

we will pick new questions that match your level based on your Timer History
Not interested in getting valuable practice questions and articles delivered to your email? No problem, unsubscribe here.
Close
Request Expert Reply
Confirm Cancel
SORT BY:
Date
Tags:
Show Tags
Hide Tags
Math Expert
Joined: 02 Sep 2009
Posts: 93672
Own Kudos [?]: 630937 [8]
Given Kudos: 82237
Send PM
Most Helpful Reply
Math Expert
Joined: 02 Sep 2009
Posts: 93672
Own Kudos [?]: 630937 [0]
Given Kudos: 82237
Send PM
General Discussion
Director
Director
Joined: 05 Jan 2020
Status:Admitted to IIM Shillong (PGPEx 2023-24)
Affiliations: CFA Institute; ICAI; BCAS
Posts: 524
Own Kudos [?]: 574 [1]
Given Kudos: 694
Location: India
WE 2: EA to CFO (Consumer Products)
GPA: 3.78
WE:Corporate Finance (Commercial Banking)
Send PM
Quant Chat Moderator
Joined: 22 Dec 2016
Posts: 3131
Own Kudos [?]: 4645 [1]
Given Kudos: 1857
Location: India
Concentration: Strategy, Leadership
Send PM
Re: GMAT Club World Cup 2022 (DAY 3): If |a| > |b| > |c|, is a*b^3*c^3 > a [#permalink]
1
Kudos
Given

|a| > |b| > |c|

Interpretation

(The distance of a from 0) > (The distance of b from 0) > (The distance of c from 0)

Question

\(a*b^{3}*c^{3} > a * b ^ {4} * c ^{2}\)

\(a*b^{3}*c^{3} - a * b ^ {4} * c ^{2} > 0\)

\(a*b^{3}*c^{2}*(c-b) > 0\)

Statement 1

All we know from this statement is a lies to the right of b and b lies to the right of c as shown below on the number line

-----------c-----------b-----------a-----------

We do not know in which position does 0 lies, however based on the above constraint we know that zero lies in either of the positions -

-----------c--0--------b-----------a-----------

-------0---c-----------b-----------a-----------

In both case -
a = +ve
b = +ve
c - b = -ve
\(c^2\) = +ve


\(a*b^{3}*c^{2}*(c-b) > 0\)

+ve * +ve * +ve * -ve = -ve

We have a definite answer "No"

Statement 2

a + b > 0

We know that a is farther away from b with respect to 0, hence a needs to be +ve for this to hold true.

Now, with reference to the given constraints we know that (The distance of b from 0) > (The distance of c from 0), however nothing is given on their positing.

-------0---c-----------b-----------a---------
-----------c--0--------b-----------a---------
----------b-------0--c-----------a-----------
----------b--c----0--------------a-----------

In any of the four case, we will either get \(b^3\) as -ve or (c-b) as -ve

Hence, this statement is also sufficient to answer a definite No.

IMO D
Manager
Manager
Joined: 14 Jun 2020
Posts: 75
Own Kudos [?]: 128 [2]
Given Kudos: 77
Send PM
Re: GMAT Club World Cup 2022 (DAY 3): If |a| > |b| > |c|, is a*b^3*c^3 > a [#permalink]
2
Kudos
Bunuel wrote:
If \(|a| > |b| > |c|\), is \(a*b^3*c^3 > a*b^4*c^2\) ?

(1) \(a > b > c\)
(2) \(a + b > 0\)



 


This question was provided by GMAT Club
for the GMAT Club World Cup Competition

Compete, Get Better, Win prizes and more

 


Attachments

WhatsApp Image 2022-07-14 at 4.13.14 PM (3).jpeg
WhatsApp Image 2022-07-14 at 4.13.14 PM (3).jpeg [ 85.81 KiB | Viewed 840 times ]

Manager
Manager
Joined: 30 Nov 2018
Posts: 139
Own Kudos [?]: 149 [1]
Given Kudos: 79
Location: India
Concentration: Strategy, Marketing
GPA: 4
Send PM
Re: GMAT Club World Cup 2022 (DAY 3): If |a| > |b| > |c|, is a*b^3*c^3 > a [#permalink]
1
Kudos
Given |a|>|b|>|c|

is a∗b^3∗c^3 > a∗b^4∗c^ 2?


Statement 1 : a > b > c (given!)


Also, |a|>|b|>|c|

Here we can say that a, b and c are surely positive.

Therefore, question a∗b^3∗c^3 > a∗b^4∗c^ 2 [in this case] can be rewritten as:

Is c>b ?? to which the answer is a 100% NO

Statement A (Sufficient)


Statement 2 : a +b>0

[also it is given |a|>|b|>|c|]. Here we can confidently say that a is positive.

In this case the question can be rewritten as : is b^3∗c^3 > b^4∗c^ 2?


We know that b^4∗c^ 2 ie (RHS) will always be positive since the powers of both b and c are even. Moreover, the expression b^3∗c^3 will always be less than b^4∗c^ 2 since |b|>|c| and power of b is less in LHS than in RHS.

Hence, statement B is sufficient too.


IMO Option D
Senior Manager
Senior Manager
Joined: 28 May 2022
Status:Dreaming and Working
Affiliations: None
Posts: 440
Own Kudos [?]: 387 [1]
Given Kudos: 83
WE:Brand Management (Manufacturing)
Send PM
Re: GMAT Club World Cup 2022 (DAY 3): If |a| > |b| > |c|, is a*b^3*c^3 > a [#permalink]
1
Kudos
Bunuel wrote:
If \(|a| > |b| > |c|\), is \(a*b^3*c^3 > a*b^4*c^2\) ?

(1) \(a > b > c\)
(2) \(a + b > 0\)

 


This question was provided by GMAT Club
for the GMAT Club World Cup Competition

Compete, Get Better, Win prizes and more

 



Given information: \(|a| > |b| > |c|\)
To determine if: \(a*b^3*c^3 > a*b^4*c^2\)?

(1) \(a > b > c\)

So, a>b>c as well as |a|>|b|>|c|. Based on these 2 conditions, we can infer the following:
1) a cannot be negative (because if a=-1, and a>b>c, then b=-2 and c=-3 for example but then their absolute value inequality condition will not satisfy 1<2<3, not the other way around)
2) b cannot be negative (same reason as above)
3) c can be negative
4) a,b,c can all be positive fractions (negative fractions will not satisfy the original inequality condition of the question stem)

Now that we have established certain possibilities regarding values, we can test values:

a b c \(a*b^3*c^3\) \(a*b^4*c^2\) \(a*b^3*c^3 > a*b^4*c^2\)
3 2 1 24 48 NO
1/2 1/3 1/4 1/3456 1/2592 NO

We can test other similar patterns but result will be the same and we have tested both integers as well as fractions
Negative a,b are not possible and having c as negative will only make \(a*b^3*c^3\) as NEGATIVE and the other one as POSITIVE so again it will be a NO as to if greater or not, hence no point checking that with values
Negative fractions will not satisfy original inequality condition

SUFFICIENT

2) \(a + b > 0\)

So, a+b>0 as well as |a|>|b|>|c|. Based on these 2 conditions, we can infer the following:

1) a cannot be negative (because if a is negative, then b will have to be greater than a for a+b>0 to hold true and that will go against the original inequality condition of the question)
2) b and c can be negative as long as they satisfy original inequality condition

Let us test using some values:

a b c \(a*b^3*c^3\) \(a*b^4*c^2\) \(a*b^3*c^3 > a*b^4*c^2\)
3 2 1 24 48 NO
1/2 1/3 1/4 1/3456 1/2592 NO
5 -3 -2 1080 1620 NO
1 -1/2 -1/3 1/216 1/144 NO

Only b or c as negative is one case not tested with values because it will lead to \(a*b^3*c^3\) as negative and then again NO to the question

SUFFICIENT

Answer - D
GMAT Club Bot
Re: GMAT Club World Cup 2022 (DAY 3): If |a| > |b| > |c|, is a*b^3*c^3 > a [#permalink]
Moderator:
Math Expert
93669 posts